Quantcast
  • Register
PhysicsOverflow is a next-generation academic platform for physicists and astronomers, including a community peer review system and a postgraduate-level discussion forum analogous to MathOverflow.

Welcome to PhysicsOverflow! PhysicsOverflow is an open platform for community peer review and graduate-level Physics discussion.

Please help promote PhysicsOverflow ads elsewhere if you like it.

News

PO is now at the Physics Department of Bielefeld University!

New printer friendly PO pages!

Migration to Bielefeld University was successful!

Please vote for this year's PhysicsOverflow ads!

Please do help out in categorising submissions. Submit a paper to PhysicsOverflow!

... see more

Tools for paper authors

Submit paper
Claim Paper Authorship

Tools for SE users

Search User
Reclaim SE Account
Request Account Merger
Nativise imported posts
Claim post (deleted users)
Import SE post

Users whose questions have been imported from Physics Stack Exchange, Theoretical Physics Stack Exchange, or any other Stack Exchange site are kindly requested to reclaim their account and not to register as a new user.

Public \(\beta\) tools

Report a bug with a feature
Request a new functionality
404 page design
Send feedback

Attributions

(propose a free ad)

Site Statistics

205 submissions , 163 unreviewed
5,047 questions , 2,200 unanswered
5,345 answers , 22,709 comments
1,470 users with positive rep
816 active unimported users
More ...

  Noether currents in QFT

+ 6 like - 0 dislike
1249 views

I am trying to organize my knowledge of Noether's theorem in QFT. There are several questions I would like to have an answer to.

In classical field theory, Noether's theorem states that for each continuous global symmetry of the action there is a corresponding current (Noether current) $j^{\mu}$, which satisfies (classically) the conservation condition:

$$ \partial_{\mu} j^{\,\mu} \simeq 0, $$

where I use the $\simeq$ sign to denote that the equation is only valid on-shell, i.e. on field configurations which are subject to the classical equations of motion.

Conserved currents lead eventually to conserved charges, which are given by

$$ Q(t) = \int d^{n-1}x \; j^{\,0}(x, t) \simeq \text{const}. $$

  1. Is it correct that these charges form an algebra (with an algebraic Lie bracket given by the Poisson bracket), which is exactly the Lie algebra of the symmetry group?

  2. Vector fields on the space-time manifold also have a Lie algebra structure given by the Lie derivative. I did some calculations and it turned out that conserved vector fields are algebraically close and therefore form a subalgebra. My question is: do Noether currents of an arbitrary field theory also form a subalgebra through the Lie derivative, and if they do, does this subalgebra have any physical meaning?

Despite the questions, this part is relatively clear. Now comes the quantum magic. In the path integral formalism, the Ward identity is a formal analogue of the classical Noether's theorem.

Noether currents are considered very important components of the quantum theory, a kind of vicars of symmetries on the physical system. I never understood this completely. For example, they are ought to be well-defined in the quantum sense and therefore are subject to normal-ordering. This sometimes leads to the modification (!) of the symmetry algebra itself, the best example being the Witt algebra of conformal symmetries and its quantum counterpart, the Virasoro algebra of the normal-ordered conformal currents.

  1. So why are currents more fundamental than geometrical symmetries of classical configurations itself? Why are they even have to be well-defined in the quantum theory, where the only observable quantities are correlations?

P.S. I have studied lots of literature, and all explanations seemed to me unclear and speculative. So I am not looking for a reference, but rather for some kind of a paraphrasing which would make things clearer.

@JamalS, I am keeping the string-theory tag because I expect string theorists to know a lot about (mentioned in my question) Witt/Virasoro algebras and therefore to give expanded answers.

This post imported from StackExchange Physics at 2015-07-11 08:01 (UTC), posted by SE-user Hindsight
asked Feb 10, 2015 in Theoretical Physics by Hindsight (90 points) [ no revision ]

1 Answer

+ 2 like - 0 dislike

Comments to the question (v5):

  1. If an action functional $S$ is invariant under a Lie algebra $L$ of symmetries, the corresponding Noether currents & charges do not always form a representation of the Lie algebra $L$. There could be (classical) anomalies. In some cases such (classical) anomalies appear as central extensions, cf. e.g. Ref. 1-3 and this Phys.SE post.

  2. The negative conclusion for point 1 is true even for Hamiltonian formulations where a Poisson bracket is defined.

  3. The 3rd question seems like an analogue of the faulty logic that requires a quantum theory to be explained in a classical language, and not vice-versa.

References:

  1. F. Toppan, On anomalies in classical dynamical systems, J. Nonlin. Math. Phys. 8 (2001) 518, arXiv:math-ph/0105051.

  2. Tomas Brauner, Spontaneous Symmetry Breaking and Nambu-Goldstone Bosons in Quantum Many-Body Systems, Symmetry 2 (2010) 609, arXiv:1001.5212; page 6-7.

  3. J.D. Brown and M. Henneaux, Central charges in the canonical realization of asymptotic symmetries: an example from three-dimensional gravity, Commun. Math. Phys. 104 (1986) 207.

This post imported from StackExchange Physics at 2015-07-11 08:01 (UTC), posted by SE-user Qmechanic
answered Feb 10, 2015 by Qmechanic (3,120 points) [ no revision ]
Quantum theory, in the path integral formulation, requires classical degrees of freedom and classical action.

This post imported from StackExchange Physics at 2015-07-11 08:02 (UTC), posted by SE-user Hindsight
Also, geometric symmetries (like conformal transformations) do not require any degrees of freedom at all. They are as much classical as they are quantum. But in classical theory, we respect them instead of modifying the algebra with central charges.

This post imported from StackExchange Physics at 2015-07-11 08:02 (UTC), posted by SE-user Hindsight

Your answer

Please use answers only to (at least partly) answer questions. To comment, discuss, or ask for clarification, leave a comment instead.
To mask links under text, please type your text, highlight it, and click the "link" button. You can then enter your link URL.
Please consult the FAQ for as to how to format your post.
This is the answer box; if you want to write a comment instead, please use the 'add comment' button.
Live preview (may slow down editor)   Preview
Your name to display (optional):
Privacy: Your email address will only be used for sending these notifications.
Anti-spam verification:
If you are a human please identify the position of the character covered by the symbol $\varnothing$ in the following word:
p$\hbar$ysicsO$\varnothing$erflow
Then drag the red bullet below over the corresponding character of our banner. When you drop it there, the bullet changes to green (on slow internet connections after a few seconds).
Please complete the anti-spam verification




user contributions licensed under cc by-sa 3.0 with attribution required

Your rights
...